www.vorhilfe.de
Vorhilfe

Kostenlose Kommunikationsplattform für gegenseitige Hilfestellungen.
Hallo Gast!einloggen | registrieren ]
Startseite · Forum · Wissen · Kurse · Mitglieder · Team · Impressum
Forenbaum
^ Forenbaum
Status Englisch
  Status Grammatik
  Status Lektüre
  Status Korrekturlesen
  Status Übersetzung
  Status Sonstiges (Englisch)

Gezeigt werden alle Foren bis zur Tiefe 2

Navigation
 Startseite...
 Neuerdings beta neu
 Forum...
 vorwissen...
 vorkurse...
 Werkzeuge...
 Nachhilfevermittlung beta...
 Online-Spiele beta
 Suchen
 Verein...
 Impressum
Das Projekt
Server und Internetanbindung werden durch Spenden finanziert.
Organisiert wird das Projekt von unserem Koordinatorenteam.
Hunderte Mitglieder helfen ehrenamtlich in unseren moderierten Foren.
Anbieter der Seite ist der gemeinnützige Verein "Vorhilfe.de e.V.".
Partnerseiten
Weitere Fächer:

Open Source FunktionenplotterFunkyPlot: Kostenloser und quelloffener Funktionenplotter für Linux und andere Betriebssysteme
Forum "Uni-Analysis" - Intervallschachtelung
Intervallschachtelung < Analysis < Hochschule < Mathe < Vorhilfe
Ansicht: [ geschachtelt ] | ^ Forum "Uni-Analysis"  | ^^ Alle Foren  | ^ Forenbaum  | Materialien

Intervallschachtelung: Frage (reagiert)
Status: (Frage) reagiert/warte auf Reaktion Status 
Datum: 20:12 Mo 07.11.2005
Autor: Franzie

Hallo an alle Mathematiker!

ich brauch mal ein paar tipps, wie ich an folgenden beweis herangehen soll:
ich soll beweisen, dass zu jeder intervallschachtelung  [mm] I_{n}genau [/mm] eine reelle Zahl a gibt mit a  [mm] \in I_{n} [/mm] für alle n [mm] \in \IN. [/mm]
wie geh ich bloß an einen solchen beweis heran?

        
Bezug
Intervallschachtelung: Antwort
Status: (Antwort) fertig Status 
Datum: 20:58 Mo 07.11.2005
Autor: angela.h.b.

Hallo,

> ich brauch mal ein paar tipps, wie ich an folgenden beweis
> herangehen soll:
>  ich soll beweisen, dass zu jeder intervallschachtelung  
> [mm]I_{n}genau[/mm] eine reelle Zahl a gibt mit a  [mm]\in I_{n}[/mm] für
> alle n [mm]\in \IN.[/mm]
>  wie geh ich bloß an einen solchen beweis
> heran?

Schritt 1 wäre, daß Du Dir darüber klar wirst, was eine Intervallschachtelung ist.
Wie habt Ihr das definiert?

Schritt2: Wie sieht so eine Schachtelung anschaulich aus?
Ist Dir anschaulich-intuitiv klar, daß es da ein a gibt, welches in jedem der Intervalle [mm] I_n [/mm] liegt?
Und wenn das ganze in Gedanken bis hierhin durchdrungen und verstanden hast, dann kannst Du darangehen und es formal beweisen.

Schritt 3: Mein Tip ist, die Folge der oberen und unteren Intervallgrenzen zu betrachten, bzw. zu zeigen daß sie konvergieren, und zwar gegen einen gemeinsamen Wert.

Gruß v. Angela




Bezug
                
Bezug
Intervallschachtelung: Rückfrage mit Korrektur
Status: (Frage) beantwortet Status 
Datum: 17:50 Di 08.11.2005
Autor: Franzie

also ich hab mir jetzt zunächst bildlich dargestellt, was eine intervallschachtelung eigentlich bedeutet und hab dann an meiner skizze erkenntlich gemacht, dass ober-und unterfolge jeweils gegen den gleichen grenzwert streben, nämlich gegen mein a.

nun mein beweis: die folgen [mm] a_{n} [/mm] und [mm] b_{n} [/mm] sind jeweils monoton und beschränkt, da sie ja in [a1,b1] liegen. nach dem monotonieprinzip über grenzwerte existieren daher die grenzwerte lim [mm] a_{n}:=a [/mm] und lim [mm] b_{n}:=b. [/mm] weiterhin gilt, dass [mm] (b_{n}-a_{n}) [/mm] sowohl gegen b-a als auch gegen 0 strebt, damit muss a=b sein. und weilbfür alle n gilt [mm] a_{n}\le [/mm] sup [mm] a_{k}=a=b=inf b_{k} \le b_{n}, [/mm] liegt a in jedem intervall [mm] [a_{n}, b_{n}] [/mm]
q.e.d.

ist das so in ordnung?
lieben gruß

Bezug
                        
Bezug
Intervallschachtelung: Antwort
Status: (Antwort) fertig Status 
Datum: 19:38 Di 08.11.2005
Autor: SEcki


> also ich hab mir jetzt zunächst bildlich dargestellt, was
> eine intervallschachtelung eigentlich bedeutet und hab dann
> an meiner skizze erkenntlich gemacht, dass ober-und
> unterfolge jeweils gegen den gleichen grenzwert streben,
> nämlich gegen mein a.

Wichtiger Punkt: wie habt ihr Vollständigkeit definiert? Also du hast jedenfalls zur Verfügung, dass monotone, beschränkte Folgen einen Grenzwert haben? Okay.

> nun mein beweis: die folgen [mm]a_{n}[/mm] und [mm]b_{n}[/mm] sind jeweils
> monoton und beschränkt, da sie ja in [a1,b1] liegen.

... und monoton da die Intervalle in einander geschachtelt sind.

> nach
> dem monotonieprinzip über grenzwerte existieren daher die
> grenzwerte lim [mm]a_{n}:=a[/mm] und lim [mm]b_{n}:=b.[/mm] weiterhin gilt,

Ja.

> dass [mm](b_{n}-a_{n})[/mm] sowohl gegen b-a als auch gegen 0

Warum gegen 0? Das musst du begründen. Alternativ kannst du ancher zeigen, dass es maximal ein Element in der Intervallschachtelung gibt.

> strebt, damit muss a=b sein. und weilbfür alle n gilt
> [mm]a_{n}\le[/mm] sup [mm]a_{k}=a=b=inf b_{k} \le b_{n},[/mm] liegt a in
> jedem intervall [mm][a_{n}, b_{n}][/mm]

Okay - aber warum gibt es nicht mehrere Punkte in der Schachtelung, zN geb ich dir mal die "Intervallschachtelung" [m][-\bruch{1}{n},1][/m]. Was ist hier faul?

> ist das so in ordnung?

Noch nicht, aber bald.

Zusatzfrage: warum müssen die Intervalle abgeschlossen sein? Gilt das Prinzip denn noch wenn man offene / halboffene Intervalle nimmt?

SEcki

Bezug
                                
Bezug
Intervallschachtelung: Rückfrage
Status: (Frage) beantwortet Status 
Datum: 20:15 Di 08.11.2005
Autor: Franzie

wenn gilt [mm] a_{n} [/mm] < [mm] b_{n} [/mm] und es existiert ein n element natürl. zahlen und es gilt lim [mm] (a_{n} [/mm] - [mm] b_{n})=0 [/mm] d.h. beide folgen konvergieren gegen den gleichen grenzwert a
(oder wie war das gemeint, ich soll erklären, weshalb sie gegen 0 streben?)
es gilt weiterhin [mm] [a_{n+1} ,b_{n+1}] \subset [a_{n}, b_{n}] [/mm]
dann gilt [mm] \cap [a_{n}, b_{n}] [/mm] ={a}
d.h. der durchschnitt aller intervalle [mm] [a_{n}, b_{n}] [/mm]
ist eine menge mit einem einzigem element a

reicht das zur klärung der restlichen lücken in meinem beweis?


Bezug
                                        
Bezug
Intervallschachtelung: Antwort
Status: (Antwort) fertig Status 
Datum: 08:51 Mi 09.11.2005
Autor: angela.h.b.

Hallo Franzi,
das, was Du da schreibst, finde ich von der Sprache her sehr schwer zu verstehen.
Es ist überhaupt keine Aussage.

> wenn gilt [mm]a_{n}[/mm] < [mm]b_{n}[/mm] und es existiert ein n element
> natürl. zahlen und es gilt lim [mm](a_{n}[/mm] - [mm]b_{n})=0[/mm]

DASS es gilt, ist aber doch genau das, was bisher unklar ist und begründet werden soll.

d.h. beide

> folgen konvergieren gegen den gleichen grenzwert a


>  (oder wie war das gemeint, ich soll erklären, weshalb sie
> gegen 0 streben?)
>  es gilt weiterhin [mm][a_{n+1} ,b_{n+1}] \subset [a_{n}, b_{n}][/mm]
>  
> dann gilt [mm]\cap [a_{n}, b_{n}][/mm] ={a}

Warum? Warum besteht der Durchschnitt nicht aus zwei Elementen?
Dies ist die Stelle, an der Du weiterarbeiten arbeiten mußt, um zum Ziel zu kommen.
Schau Dir nochmal Seckis Beitag an.

>  d.h. der durchschnitt aller intervalle [mm][a_{n}, b_{n}][/mm]
>  ist
> eine menge mit einem einzigem element a
>  
> reicht das zur klärung der restlichen lücken in meinem
> beweis?

Nein, leider noch nicht.

Gruß v. Angela


Bezug
                                
Bezug
Intervallschachtelung: Rückfrage
Status: (Frage) beantwortet Status 
Datum: 13:12 Mi 09.11.2005
Autor: Franzie

also ich brauch echt nochmal einen anstoß, wie ich beweisen soll, dass beide folgen gegen den gleichen grenzwert streben und dass a das einzige element darin ist. ich weiß echt nicht, wie ich das anstellen soll.
außerdem muss ich ja noch begründen, warum  [mm] (a_{n}-b_{n}) [/mm] sowohl gegen b-a als auch gegen 0 strebt. könnt ihr mir da weiter helfen?

Bezug
                                        
Bezug
Intervallschachtelung: Antwort
Status: (Antwort) fertig Status 
Datum: 17:21 Mi 09.11.2005
Autor: SEcki


>  außerdem muss ich ja noch begründen, warum  [mm](a_{n}-b_{n})[/mm]
> sowohl gegen b-a als auch gegen 0 strebt. könnt ihr mir da
> weiter helfen?

Ja - welche Eigenschaft der Intervallschachtelung hast du noch nicht benutzt? Es ist eher banal, aber es sollte quasi eine geforderte Eigenschaft sein, dass [m]|a_n-b_n|[/m] gegen Null geht. Warum gibt es jetzt maximal ein Element im Schnitt? Was wäre wenn es denn zwei geben würde - mit dem Abstand?

SEcki

Bezug
                                                
Bezug
Intervallschachtelung: Rückfrage mit Korrektur
Status: (Frage) beantwortet Status 
Datum: 18:42 Mi 09.11.2005
Autor: Franzie

okay, also ich hab grad noch die letzte geforderte eigenschaft entdeckt. wenn wir die in der vorlesung so definiert haben, dass [mm] lim(a_{n}-a_{n})=0 [/mm] ist, brauch ich das ja nicht nochmal extra erklären.
aber nun zur einzigkeit der zahl a:
ich hab mir das so überlegt, wenn es angenommen zwei zahlen a und b geben würde (wobei a<b), dann läge das intervall [a,b] ja in jedem intervall und jedes intervall hätte eine länge (das meintest du doch sicher mit dem abstand?) [mm] \ge [/mm] b-a. aber das wäre ein widerspruch zu meiner eigenschaft, dass lim(b-a)=0 ist und damit kann es nur eine zahl a geben, die in allen intervallen liegt.
okay so?

lieber gruß und danke für die nützlichen tipps!

Bezug
                                                        
Bezug
Intervallschachtelung: Antwort
Status: (Antwort) fertig Status 
Datum: 18:52 Mi 09.11.2005
Autor: SEcki


> okay, also ich hab grad noch die letzte geforderte
> eigenschaft entdeckt. wenn wir die in der vorlesung so
> definiert haben, dass [mm]lim(a_{n}-a_{n})=0[/mm] ist, brauch ich
> das ja nicht nochmal extra erklären.

Abgesehen davon, dass das eine a wohl ein b ist, stimmt das schon. Aber du solltest sowas halt hinschreiben - bla folgt aus blubber.

>  aber nun zur einzigkeit der zahl a:
>  ich hab mir das so überlegt, wenn es angenommen zwei
> zahlen a und b geben würde (wobei a<b), dann läge das
> intervall [a,b] ja in jedem intervall und jedes intervall
> hätte eine länge (das meintest du doch sicher mit dem
> abstand?) [mm]\ge[/mm] b-a. aber das wäre ein widerspruch zu meiner
> eigenschaft, dass lim(b-a)=0 ist und damit kann es nur eine
> zahl a geben, die in allen intervallen liegt.

Ja, maximal eine - warum mindestens eine drin liegt, hast du ja schon gezeigt. Aber du solltest wirklich die Zusatzaufgabe machen: geht das Intervallschachtelungsprinzip auch mit halboffenen, offenen Intervallen?

SEcki

Bezug
                                                                
Bezug
Intervallschachtelung: Rückfrage
Status: (Frage) reagiert/warte auf Reaktion Status 
Datum: 19:24 Mi 09.11.2005
Autor: Franzie

bis jetzt haben wir eigentlich immer nur von abgeschlossenen intervallen geredet. gilt denn dieser satz überhaupt für offene intervalle? in meinem buch ist hier lediglich von geschlossenen intervallen die rede.


Bezug
                                                                        
Bezug
Intervallschachtelung: Mitteilung
Status: (Mitteilung) Reaktion unnötig Status 
Datum: 20:17 Mi 09.11.2005
Autor: SEcki


> bis jetzt haben wir eigentlich immer nur von
> abgeschlossenen intervallen geredet. gilt denn dieser satz
> überhaupt für offene intervalle? in meinem buch ist hier
> lediglich von geschlossenen intervallen die rede.

Das sollst du ja eben herausfinden! Das ist jetzt deine Aufgabe. :-)

SEcki

Bezug
Ansicht: [ geschachtelt ] | ^ Forum "Uni-Analysis"  | ^^ Alle Foren  | ^ Forenbaum  | Materialien


^ Seitenanfang ^
www.englischraum.de
[ Startseite | Forum | Wissen | Kurse | Mitglieder | Team | Impressum ]